If $R$ is uniform $(0,1)$ and $Psi$ is uniform $(0,2pi)$, what is the density law of $(X,Y)=(RcosPsi...












3














If $R$ is uniform $(0,1)$ and $Psi$ is uniform $(0,2pi)$, what is the density law of $(X,Y)=(RcosPsi ,RsinPsi)$ ?



I tried as follow :



I separate the cases where $Psiin (0,pi/2)$, $Psiin (pi/2,pi)$, $Psiin (pi,3pi/2)$ and $Psiin (3pi/2,2pi)$.




  • For the first one, $(x,y)=g(r,psi )=(rcospsi ,rsinpsi )$ implies that $(r,psi )=h(x,y)=(sqrt{x^2+y^2},arctan(y/x))$ and thus $$f_{X,Y}(x,y)=f_{R, Psi}(h(x,y))|J_{h}(x,y)|, if x^2+y^2leq 1, x,y>0,$$


  • For the second one, $(x,y)=k(r,psi)=(rcospsi,rsinpsi)$ implies $(r,psi)=ell(x,y)=(sqrt{x^2+y^2}, pi-arctan(y/x))$, and thus $$f_{X,Y}(x,y)=f_{R,psi}(ell(x,y))J_{ell}(x,y), if x^2+y^2leq 1, x<0, y<0.$$



But now, how can I know $f_{X,Y}(x,y)$ when $x=0$ ? i.e. what is $f_{X,Y}(0,y)$ for $yin [0,1]$ ? Because I didn't considered in my manipulation since I avoid $psi=pi/2$. I can compute $F_{X,Y}(0,y)$, but to get $f_{X,Y}(0,y)$ I can't derivate, so I'm a bit in truble.










share|cite|improve this question






















  • @LeeDavidChungLin : You should really read the question to the end, and you'll see that it's absolutely not duplicate ;)
    – NewMath
    Nov 30 '18 at 13:33










  • Presumably, $R$ and $Psi$ are independent.
    – StubbornAtom
    Nov 30 '18 at 14:03
















3














If $R$ is uniform $(0,1)$ and $Psi$ is uniform $(0,2pi)$, what is the density law of $(X,Y)=(RcosPsi ,RsinPsi)$ ?



I tried as follow :



I separate the cases where $Psiin (0,pi/2)$, $Psiin (pi/2,pi)$, $Psiin (pi,3pi/2)$ and $Psiin (3pi/2,2pi)$.




  • For the first one, $(x,y)=g(r,psi )=(rcospsi ,rsinpsi )$ implies that $(r,psi )=h(x,y)=(sqrt{x^2+y^2},arctan(y/x))$ and thus $$f_{X,Y}(x,y)=f_{R, Psi}(h(x,y))|J_{h}(x,y)|, if x^2+y^2leq 1, x,y>0,$$


  • For the second one, $(x,y)=k(r,psi)=(rcospsi,rsinpsi)$ implies $(r,psi)=ell(x,y)=(sqrt{x^2+y^2}, pi-arctan(y/x))$, and thus $$f_{X,Y}(x,y)=f_{R,psi}(ell(x,y))J_{ell}(x,y), if x^2+y^2leq 1, x<0, y<0.$$



But now, how can I know $f_{X,Y}(x,y)$ when $x=0$ ? i.e. what is $f_{X,Y}(0,y)$ for $yin [0,1]$ ? Because I didn't considered in my manipulation since I avoid $psi=pi/2$. I can compute $F_{X,Y}(0,y)$, but to get $f_{X,Y}(0,y)$ I can't derivate, so I'm a bit in truble.










share|cite|improve this question






















  • @LeeDavidChungLin : You should really read the question to the end, and you'll see that it's absolutely not duplicate ;)
    – NewMath
    Nov 30 '18 at 13:33










  • Presumably, $R$ and $Psi$ are independent.
    – StubbornAtom
    Nov 30 '18 at 14:03














3












3








3







If $R$ is uniform $(0,1)$ and $Psi$ is uniform $(0,2pi)$, what is the density law of $(X,Y)=(RcosPsi ,RsinPsi)$ ?



I tried as follow :



I separate the cases where $Psiin (0,pi/2)$, $Psiin (pi/2,pi)$, $Psiin (pi,3pi/2)$ and $Psiin (3pi/2,2pi)$.




  • For the first one, $(x,y)=g(r,psi )=(rcospsi ,rsinpsi )$ implies that $(r,psi )=h(x,y)=(sqrt{x^2+y^2},arctan(y/x))$ and thus $$f_{X,Y}(x,y)=f_{R, Psi}(h(x,y))|J_{h}(x,y)|, if x^2+y^2leq 1, x,y>0,$$


  • For the second one, $(x,y)=k(r,psi)=(rcospsi,rsinpsi)$ implies $(r,psi)=ell(x,y)=(sqrt{x^2+y^2}, pi-arctan(y/x))$, and thus $$f_{X,Y}(x,y)=f_{R,psi}(ell(x,y))J_{ell}(x,y), if x^2+y^2leq 1, x<0, y<0.$$



But now, how can I know $f_{X,Y}(x,y)$ when $x=0$ ? i.e. what is $f_{X,Y}(0,y)$ for $yin [0,1]$ ? Because I didn't considered in my manipulation since I avoid $psi=pi/2$. I can compute $F_{X,Y}(0,y)$, but to get $f_{X,Y}(0,y)$ I can't derivate, so I'm a bit in truble.










share|cite|improve this question













If $R$ is uniform $(0,1)$ and $Psi$ is uniform $(0,2pi)$, what is the density law of $(X,Y)=(RcosPsi ,RsinPsi)$ ?



I tried as follow :



I separate the cases where $Psiin (0,pi/2)$, $Psiin (pi/2,pi)$, $Psiin (pi,3pi/2)$ and $Psiin (3pi/2,2pi)$.




  • For the first one, $(x,y)=g(r,psi )=(rcospsi ,rsinpsi )$ implies that $(r,psi )=h(x,y)=(sqrt{x^2+y^2},arctan(y/x))$ and thus $$f_{X,Y}(x,y)=f_{R, Psi}(h(x,y))|J_{h}(x,y)|, if x^2+y^2leq 1, x,y>0,$$


  • For the second one, $(x,y)=k(r,psi)=(rcospsi,rsinpsi)$ implies $(r,psi)=ell(x,y)=(sqrt{x^2+y^2}, pi-arctan(y/x))$, and thus $$f_{X,Y}(x,y)=f_{R,psi}(ell(x,y))J_{ell}(x,y), if x^2+y^2leq 1, x<0, y<0.$$



But now, how can I know $f_{X,Y}(x,y)$ when $x=0$ ? i.e. what is $f_{X,Y}(0,y)$ for $yin [0,1]$ ? Because I didn't considered in my manipulation since I avoid $psi=pi/2$. I can compute $F_{X,Y}(0,y)$, but to get $f_{X,Y}(0,y)$ I can't derivate, so I'm a bit in truble.







probability






share|cite|improve this question













share|cite|improve this question











share|cite|improve this question




share|cite|improve this question










asked Nov 30 '18 at 12:35









NewMathNewMath

3808




3808












  • @LeeDavidChungLin : You should really read the question to the end, and you'll see that it's absolutely not duplicate ;)
    – NewMath
    Nov 30 '18 at 13:33










  • Presumably, $R$ and $Psi$ are independent.
    – StubbornAtom
    Nov 30 '18 at 14:03


















  • @LeeDavidChungLin : You should really read the question to the end, and you'll see that it's absolutely not duplicate ;)
    – NewMath
    Nov 30 '18 at 13:33










  • Presumably, $R$ and $Psi$ are independent.
    – StubbornAtom
    Nov 30 '18 at 14:03
















@LeeDavidChungLin : You should really read the question to the end, and you'll see that it's absolutely not duplicate ;)
– NewMath
Nov 30 '18 at 13:33




@LeeDavidChungLin : You should really read the question to the end, and you'll see that it's absolutely not duplicate ;)
– NewMath
Nov 30 '18 at 13:33












Presumably, $R$ and $Psi$ are independent.
– StubbornAtom
Nov 30 '18 at 14:03




Presumably, $R$ and $Psi$ are independent.
– StubbornAtom
Nov 30 '18 at 14:03










1 Answer
1






active

oldest

votes


















1














Looking at the area element $dA$ of the unit disk, we see that
$$dA = rdrdpsi = dxdy $$



enter image description here



Hence
$$f_{X, Y}left(x, yright)dxdy = f_{R, Psi}left(r, psiright)drdpsi $$
$$f_{X, Y}left(x, yright) = dfrac{f_{R, Psi}left(r, psiright)}{r} $$
$$f_{X, Y}left(x, yright) = dfrac{1}{2pisqrt{x^{2} + y^{2}}}, 0 < x^{2} + y^{2} < 1 $$






share|cite|improve this answer





















    Your Answer





    StackExchange.ifUsing("editor", function () {
    return StackExchange.using("mathjaxEditing", function () {
    StackExchange.MarkdownEditor.creationCallbacks.add(function (editor, postfix) {
    StackExchange.mathjaxEditing.prepareWmdForMathJax(editor, postfix, [["$", "$"], ["\\(","\\)"]]);
    });
    });
    }, "mathjax-editing");

    StackExchange.ready(function() {
    var channelOptions = {
    tags: "".split(" "),
    id: "69"
    };
    initTagRenderer("".split(" "), "".split(" "), channelOptions);

    StackExchange.using("externalEditor", function() {
    // Have to fire editor after snippets, if snippets enabled
    if (StackExchange.settings.snippets.snippetsEnabled) {
    StackExchange.using("snippets", function() {
    createEditor();
    });
    }
    else {
    createEditor();
    }
    });

    function createEditor() {
    StackExchange.prepareEditor({
    heartbeatType: 'answer',
    autoActivateHeartbeat: false,
    convertImagesToLinks: true,
    noModals: true,
    showLowRepImageUploadWarning: true,
    reputationToPostImages: 10,
    bindNavPrevention: true,
    postfix: "",
    imageUploader: {
    brandingHtml: "Powered by u003ca class="icon-imgur-white" href="https://imgur.com/"u003eu003c/au003e",
    contentPolicyHtml: "User contributions licensed under u003ca href="https://creativecommons.org/licenses/by-sa/3.0/"u003ecc by-sa 3.0 with attribution requiredu003c/au003e u003ca href="https://stackoverflow.com/legal/content-policy"u003e(content policy)u003c/au003e",
    allowUrls: true
    },
    noCode: true, onDemand: true,
    discardSelector: ".discard-answer"
    ,immediatelyShowMarkdownHelp:true
    });


    }
    });














    draft saved

    draft discarded


















    StackExchange.ready(
    function () {
    StackExchange.openid.initPostLogin('.new-post-login', 'https%3a%2f%2fmath.stackexchange.com%2fquestions%2f3020048%2fif-r-is-uniform-0-1-and-psi-is-uniform-0-2-pi-what-is-the-density%23new-answer', 'question_page');
    }
    );

    Post as a guest















    Required, but never shown

























    1 Answer
    1






    active

    oldest

    votes








    1 Answer
    1






    active

    oldest

    votes









    active

    oldest

    votes






    active

    oldest

    votes









    1














    Looking at the area element $dA$ of the unit disk, we see that
    $$dA = rdrdpsi = dxdy $$



    enter image description here



    Hence
    $$f_{X, Y}left(x, yright)dxdy = f_{R, Psi}left(r, psiright)drdpsi $$
    $$f_{X, Y}left(x, yright) = dfrac{f_{R, Psi}left(r, psiright)}{r} $$
    $$f_{X, Y}left(x, yright) = dfrac{1}{2pisqrt{x^{2} + y^{2}}}, 0 < x^{2} + y^{2} < 1 $$






    share|cite|improve this answer


























      1














      Looking at the area element $dA$ of the unit disk, we see that
      $$dA = rdrdpsi = dxdy $$



      enter image description here



      Hence
      $$f_{X, Y}left(x, yright)dxdy = f_{R, Psi}left(r, psiright)drdpsi $$
      $$f_{X, Y}left(x, yright) = dfrac{f_{R, Psi}left(r, psiright)}{r} $$
      $$f_{X, Y}left(x, yright) = dfrac{1}{2pisqrt{x^{2} + y^{2}}}, 0 < x^{2} + y^{2} < 1 $$






      share|cite|improve this answer
























        1












        1








        1






        Looking at the area element $dA$ of the unit disk, we see that
        $$dA = rdrdpsi = dxdy $$



        enter image description here



        Hence
        $$f_{X, Y}left(x, yright)dxdy = f_{R, Psi}left(r, psiright)drdpsi $$
        $$f_{X, Y}left(x, yright) = dfrac{f_{R, Psi}left(r, psiright)}{r} $$
        $$f_{X, Y}left(x, yright) = dfrac{1}{2pisqrt{x^{2} + y^{2}}}, 0 < x^{2} + y^{2} < 1 $$






        share|cite|improve this answer












        Looking at the area element $dA$ of the unit disk, we see that
        $$dA = rdrdpsi = dxdy $$



        enter image description here



        Hence
        $$f_{X, Y}left(x, yright)dxdy = f_{R, Psi}left(r, psiright)drdpsi $$
        $$f_{X, Y}left(x, yright) = dfrac{f_{R, Psi}left(r, psiright)}{r} $$
        $$f_{X, Y}left(x, yright) = dfrac{1}{2pisqrt{x^{2} + y^{2}}}, 0 < x^{2} + y^{2} < 1 $$







        share|cite|improve this answer












        share|cite|improve this answer



        share|cite|improve this answer










        answered Nov 30 '18 at 13:41









        rzchrzch

        1363




        1363






























            draft saved

            draft discarded




















































            Thanks for contributing an answer to Mathematics Stack Exchange!


            • Please be sure to answer the question. Provide details and share your research!

            But avoid



            • Asking for help, clarification, or responding to other answers.

            • Making statements based on opinion; back them up with references or personal experience.


            Use MathJax to format equations. MathJax reference.


            To learn more, see our tips on writing great answers.





            Some of your past answers have not been well-received, and you're in danger of being blocked from answering.


            Please pay close attention to the following guidance:


            • Please be sure to answer the question. Provide details and share your research!

            But avoid



            • Asking for help, clarification, or responding to other answers.

            • Making statements based on opinion; back them up with references or personal experience.


            To learn more, see our tips on writing great answers.




            draft saved


            draft discarded














            StackExchange.ready(
            function () {
            StackExchange.openid.initPostLogin('.new-post-login', 'https%3a%2f%2fmath.stackexchange.com%2fquestions%2f3020048%2fif-r-is-uniform-0-1-and-psi-is-uniform-0-2-pi-what-is-the-density%23new-answer', 'question_page');
            }
            );

            Post as a guest















            Required, but never shown





















































            Required, but never shown














            Required, but never shown












            Required, but never shown







            Required, but never shown

































            Required, but never shown














            Required, but never shown












            Required, but never shown







            Required, but never shown







            Popular posts from this blog

            How do I know what Microsoft account the skydrive app is syncing to?

            When does type information flow backwards in C++?

            Grease: Live!